Kernel and image of linear transformation

Click For Summary
The discussion focuses on finding the Kernel and Image of the linear transformation T: R4 --> R3 defined by the matrix A. The Kernel is identified as the set of vectors v in R4 such that Av = 0, while the Image is derived from the linear combinations of the columns of A. Participants express confusion regarding the formulation of the Image and the correct representation of the basis for both the Kernel and the Image. The general solution for the Image is discussed, leading to a basis representation. Clarification on the definitions and calculations for both concepts is essential for accurate problem-solving.
Locoism
Messages
77
Reaction score
0

Homework Statement


For the linear transformation T: R4 --> R3 defined by TA: v -->Av
find a basis for the Kernel of TA and for the Image of of TA where A is
2 4 6 2
1 3 -4 1
4 10 -2 4

Homework Equations



Let v =
a1 b1 c1
a2 b2 c2
a3 b3 c3
a4 b4 c4

The Attempt at a Solution


so v is a 4x3 matrix, and Ker(T) would just be the solution for Av = 0.
I was unsure as to what the Image would be give by. Is it the matrix
2a1+4b1+6c1+2d1, 2a2+4b2+6c2+2d2, 2a3+4b3+6c3+2d3,
1a1+3b1-4c1+d1, ... etc

(just the general solution of the multiplication)
Which generalizes to
2 0 2
0 1 2
so the basis is [1, 2, -1]

How would I find a basis for the Kernel?
 
Last edited:
Physics news on Phys.org
Locoism said:

Homework Statement


For the linear transformation T: R4 --> R3 defined by TA: v -->Av
find a basis for the Kernel of TA and for the Image of of TA where A is
2 4 6 2
1 3 -4 1
4 10 -2 4


Homework Equations



Let v =
a1 b1 c1
a2 b2 c2
a3 b3 c3
a4 b4 c4
You're really heading down the wrong path here. v is a vector in R4.
Locoism said:

The Attempt at a Solution


so v is a 4x3 matrix, and Ker(T) would just be the solution for Av = 0.
Ker(T) is the set of all vectors v in R4 such that Tv = 0.
Locoism said:
I was unsure as to what the Image would be give by. Is it the matrix
2a1+4b1+6c1+2d1, 2a2+4b2+6c2+2d2, 2a3+4b3+6c3+2d3,
1a1+3b1-4c1+d1, ... etc

(just the general solution of the multiplication)
Which generalizes to
2 0 2
0 1 2
so the basis is [1, 2, -1]

How would I find a basis for the Kernel?
 
Question: A clock's minute hand has length 4 and its hour hand has length 3. What is the distance between the tips at the moment when it is increasing most rapidly?(Putnam Exam Question) Answer: Making assumption that both the hands moves at constant angular velocities, the answer is ## \sqrt{7} .## But don't you think this assumption is somewhat doubtful and wrong?

Similar threads

  • · Replies 9 ·
Replies
9
Views
2K
Replies
2
Views
3K
Replies
9
Views
15K
  • · Replies 12 ·
Replies
12
Views
5K
  • · Replies 5 ·
Replies
5
Views
2K
Replies
5
Views
2K
  • · Replies 1 ·
Replies
1
Views
2K
  • · Replies 1 ·
Replies
1
Views
8K
  • · Replies 1 ·
Replies
1
Views
1K
  • · Replies 2 ·
Replies
2
Views
2K